1. Trang chủ
  2. » Thể loại khác

MỘT SỐ VẤN ĐỀ VỀ BẤT ĐẲNG THỨC.

75 7 0

Đang tải... (xem toàn văn)

Tài liệu hạn chế xem trước, để xem đầy đủ mời bạn chọn Tải xuống

THÔNG TIN TÀI LIỆU

TRƯỜNG TRUNG HỌC PHỔ THÔNG CHUYÊN LÊ KHIẾT =====@@@===== MỘT SỐ VẤN ĐỀ VỀ BẤT ĐẲNG THỨC TÁC GIẢ: NGUYỄN ANH KHOA LỚP : 10 TOÁN QUẢNG NGÃI, THÁNG NĂM 2009 PDF created with pdfFactory Pro trial version www.pdffactory.com NGUYỄN ANH KHOA A.Lời giới thiệu: Các bất đẳng thức có điều kiện khơng có điều kiện hai tốn hồn tồn khác ẩn sau chúng lại có mối quan hệ mật thiết với Chính liên quan mật thiết làm nảy sinh kĩ thuật chứng minh bất đẳng thức “kĩ thuật chuẩn hố” Trong viết khám phá kĩ thuật có ý nghĩa nhé! B Kiến thức bản: 1.1 Bất đẳng thức nhất: Hàm số f ( x1 ; x2 ; xn ) biến x1 ; x2 ; xn gọi hàm bậc α tồn số thực t thoả mãn: f (tx1 ; tx2 ; txn ) = t α f ( x1 ; x2 ; xn ) Từ ta có định nghĩa bất đẳng thức sau: * Bất đẳng thức bất đẳng thức có dạng: f ( x1 ; x2 ; xn ) ≥ f ( x1; x2 ; xn ) hàm bậc α Ví dụ: Bất đẳng thức AM-GM bất đẳng thức bậc tx1 + tx2 + + txn ≥ n n tx1.tx2 txn ⇒ x1 + x2 + + xn ≥ n n x1 x2 xn Từ trở viết nói đến bất đẳng thức ta không cần quan tâm đến bậc α 1.2 Một số phương pháp, kĩ thuật chứng minh bất đẳng thức: Trong phần ta không xét hết tất phương pháp, kĩ thuật chứng minh bất đẳng thức từ trước tới mà xét số phương pháp áp dụng viết 1.2.1 Phương pháp dồn biến: Phương pháp dồn biến tư tưởng làm giảm số biến có thơng qua đại lượng trung bình, đưa bất đẳng thức cần chứng minh dạng đơn giản chứng minh trực tiếp cách khảo sát hàm biến Định lí dồn biến: Giả sử f ( x1 ; x2 ; xn ) hàm số liên tục đối xứng với tất biến xác định miền liên thong thoả mãn điều kiện sau: x +x x +x  f ( x1 , x2 , xn ) ≥ f  ; ; x3 ; xn  (1)   x + x + + xn Khi bất đẳng thức sau thoả mãn f ( x1 ; x2 ; xn ) ≥ f ( x; x; x ) x = n Điều kiện (1) biến đổi thành số dạng khác như: f ( x1 , x2 , xn ) ≥ f ( x1 x2 , x1 x2 , x3 , xn )  x2+x x2+x  2 f ( x1 , x2 , xn ) ≥ f  , , x3 , xn    2   Tuy nhiên viết ta ý đến phương pháp dồn biến với bất đẳng thức biến nên ta xét đến trường hợp cụ thể sau: Giả sử ta cần chứng minh: f ( x1 , x2 , x3 ) ≥ ta chứng minh: f ( x1 , x2 , xn ) ≥ f (t , t , x3 ) Trong giá trị t : PDF created with pdfFactory Pro trial version www.pdffactory.com x1 + x2 + Trung bình nhân: t = x1 x2 + Trung bình cộng: t = x12 + x2 + Trung bình bình phương: t = Sau ta cần chứng minh f (t , t , x3 ) ≥ toán giải Chú ý chứng minh bất đẳng thức có điều kiện, ta thực phép dồn biến phép dồn biến phải đảm bảo thoả mãn điều kiện biến; ví dụ cho điều kiện tổng ta dồn biến trung bình cộng mà thơi Phương pháp dồn biến mà dựa vào trung bình cộng; trung bình nhân; trung bình bình phương dạng đơn giản ngồi ta cịn có số phép dồn biến sau: + Dồn biến toàn miền : sử dụng bất đẳng thức cần chứng minh có đại lượng chênh lệch bậc đại lượng xấp xỉ ( x − y ), ( y − z ), ( z − x ) : f ( x, y , z ) ≥ f ( x − z , y − z , 0) + Dồn biến biên: sử dụng đẳng thức xảy giá trị biên f ( x, y , z ) ≥ f (0, s , t ) s,t tuỳ thuộc vào tốn + Dồn biến khơng xác định: (UMV) Nếu f hàm liên tục đối xứng xác định tập U  thoả điều kiện: f ( , xi , , x j , ) ≥   x + xj x + xj    , , i ,  ; f ( , 0, , xi + x j , )  f  , i 2    Khi với x1 , x2 , xn ∈U f ( x1 , x2 , xn ) ≥ {Ct }t = , Nghĩa GTNN f ( x1 , x2 , xn ) n −1 đạt số x1 , x2 , xn có t số 0, số lại 1.2.2 Bất đẳng thức Schur kĩ thuật đổi biến P,Q,R: a Bất đẳng thức Schur: Với số thực không âm a, b, c, k ta có: a k (a − b)(a − c) + b k (b − c )(b − a ) + c k (c − a )(c − b) ≥ Nếu k = , ta có: a(a − b)(a − c) + b(b − c )(b − a ) + c(c − a )(c − b) ≥ (1) (1) xuất dạng sau: 1.(b + a − c)(b + c − a)(a + c − b) ≤ abc 2.(a + b + c )3 + 9abc ≥ 4(a + b + c )(ab + bc + ca ) 3.a + b3 + c3 + 3abc ≥ ab(a + b) + bc(b + c) + ca (c + a) Nếu k = , ta có: a (a − b)(a − c) + b (b − c )(b − a ) + c (c − a)(c − b) ≥ (2) (2) xuất dạng sau: a + b + c + abc (a + b + c) ≥ a (b + c) + b3 (c + a) + c (a + b) * Chú ý: Bất đẳng thức Schur bậc với số thực a, b, c b Bất đẳng thức Schur suy rộng:(Vornicu-Schur) Với số thực dương a, b, c, x, y , z thoả mãn (a, b, c); ( x, y, z ) đơn điệu Khi ta có: x(a − b)(a − c) + y (b − c)(b − a) + z (c − a)(c − b) ≥ Việc chứng minh bất đẳng thức Vornicu-Schur không khác cách chứng minh bất đẳng thức Schur áp dụng lại đa dạng phong phú bất đẳng thức Schur Sau bất đẳng thức mạnh bất đẳng thức Schur công cụ chứng minh phải dùng tới phương pháp phân tích bình phương S.O.S: (bài tốn xét phần sau) a3 + b3 + c3 + 3abc ≥ ab 2(a + b2 ) + bc 2(b + c ) + ca 2(c + a ) PDF created with pdfFactory Pro trial version www.pdffactory.com c Kĩ thuật đổi biến P,Q,R: Định lí: Mọi đa thức đối xứng F (a, b, c ) biểu diễn dạng đa thức đối xứng Viete Nghĩa biểu diễn qua đại lượng a + b + c, ab + bc + ca, abc Từ ta có ý tưởng sau: Khi chứng minh bất đẳng thức đối xứng ta đổi biến lại sau: Đặt p = a + b + c; q = ab + bc + ca; r = abc Khi bất đẳng thức Schur bậc 0,1,2 biểu diễn lại sau: Với k = pq − 9r ≥ (i) Với k = (1) ⇔ p − pq + 9r ≥ (ii) Với k = (2) ⇔ p − p q + 4q + pr ≥ (iii) Trong thực hành ta thường sử dụng số kết phân tích sau: 1.ab(a + b) + bc (b + c) + ca (c + a) = pq − 3r 2.(a + b)(b + c)(c + a) = pq − r 3.(a + b)(a + c) + (b + c)(b + a) + (c + a )(c + b) = p + q 4.ab(a + b ) + bc (b + c ) + ca (c + a ) = p q − 2q − pr 5.a + b + c = p − 2q 6.a + b3 + c3 = p − pq + 3r 7.a + b4 + c = p − p q + 2q + pr 8.a 2b + b 2c + c a = q − pr 9.a 3b3 + b3c3 + c 3a = q − pqr + 3r 10.a 4b + b 4c + c a = q − pq r + p r + 4qr Điều quan trọng mà ta rút (i);(ii);(iii) ta suy ra: pq r≤  p(4q − p )  r ≥ max 0,     (4q − p )( p − q)  r ≥ max 0,  6p   Đồng thời việc chứng minh ta thường sử dụng số bất đẳng thức ràng buộc biến p,q,r : p ≥ 3q p ≥ 27r q ≥ pr p3 + 9r ≥ pq p q + pr ≥ 4q 1.2.2 Look at the end point: Đây kĩ thuật xét phần tử biên, viết ta sử dụng số định lí sau: Định lí 1: Nếu f ( x ) hàm bậc theo x f (a) ≥ 0; f (b) ≥ f ( x) ≥ với x ∈ [ a, b ] PDF created with pdfFactory Pro trial version www.pdffactory.com Định lí 2: Nếu f ( x ) hàm bậc theo x : { f (a); f (b)} ≤ f ( x ) ≤ max { f (a ); f (b)} với x ∈ [ a, b ] Định lí 3: Nếu f ( x ) hàm số lồi khoảng [ a, b ] f ( x) ≤ max { f (a), f (b)} Định lí 4: Nếu f ( x ) hàm số lõm khoảng [ a, b ] f ( x ) ≥ { f (a ), f (b)} 1.2.3 Phương pháp cân hệ số: 1.2.4 Kĩ thuật chọn điểm rơi AM-GM: 1.2.5 Phương pháp phản chứng: Đây ý tưởng hay việc chứng minh bất đẳng thức sáng tạo bất đẳng thức Phương pháp lấy ý tưởng từ toán sau: Bài toán: Cho hai hàm F ( x1 , x2 , xn ); G ( x1 , x2 , xn ) bậc α > Ta xét mệnh đề sau: Nếu F ( x1 , x2 , xn ) = k G ( x1 , x2 , xn ) ≥ m (*) ( k, m > 0) + Nếu F; G hai hàm tăng x1 , x2 , xn Khi đó: (*) ⇔ Nếu G ( x1 , x2 , xn ) = m F ( x1 , x2 , xn ) ≤ k + Nếu F hàm tăng; G hàm giảm x1 , x2 , xn Khi đó: (*) ⇔ Nếu G ( x1 , x2 , xn ) = m F ( x1 , x2 , xn ) ≥ k Bạn đọc tự chứng minh toán nên ghi nhớ kết để sau tiện sử dụng C Kĩ thuật chuẩn hoá bất đẳng thức đối xứng: Người ta sử dụng ý tưởng chuẩn hoá sau: Giả sử ta cần chứng minh bất đẳng thức f ( x1 , x2 , xn ) ≥ g ( x1 , x2 , xn ) f g hai hàm bậc tính chất hàm ta chuyển bất đẳng thức việc chứng minh bất đẳng thức f ( x1 , x2 , xn ) ≥ a với x1 , x2 , xn thoả mãn g ( x1 , x2 , xn ) = a Lợi ích việc chuẩn hố ta làm đơn giản biểu thức bất đẳng thức cần chứng minh, tận dụng số tính chất đặc biệt số Bạn đọc hiểu kĩ thuật chuẩn hố thơng qua tốn sau ab + bc + ca (a + b)(b + c )(c + a ) ≤ (1) Problem: (STBĐT) CMR với a,b,c khơng âm Chắc hẳn bạn điều nhận toán từ sách “ Sáng tạo bất đẳng thức” anh Phạm Kim Hùng phần anh Hùng giới thiệu kĩ thuật chuẩn hố Vì tơi khơng đưa lời giải mà quan tâm tới cách thức chuẩn hoá, lại chuẩn hố Hiển nhiên bạn điều dễ dàng nhận bất đẳng thức cần chứng minh a b c Theo sách, anh Hùng chuẩn hoá ab + bc + ca = Khi ta lấy a ' = ; b ' = ; c ' = ta cần chọn t cho t t t ab + bc + ca a 'b ' + b 'c ' + c ' a ' = lúc ta tìm t = Bât đẳng thức với a ' , b ' , c ' nên ' ' ' với a,b,c sau nhân a , b , c với t Như việc tìm số t xong ( tất nhiên bước ta làm nháp không cần ghi vào làm).Bây ta coi chưa biết số t, ta tạo điều kiện a,b,c sau PDF created with pdfFactory Pro trial version www.pdffactory.com (a + b)(b + c )(c + a ) (ab + bc + ca)3 ≥1 : 27 (a + b)2 (b + c) (c + a )2 ⇔  ab + bc + ca  64.    ≥1 2           a b b c c a      ⇔3 + + +  ab + bc + ca ab + bc + ca   ab + bc + ca ab + bc + ca   ab + bc + ca ab + bc + ca      3 3 3      a b c Đặt x = ;y= ;z = ta có điều kiện xy + yz + xz = ab + bc + ca ab + bc + ca ab + bc + ca 3 bất đẳng thức cần chứng minh trở thành : ( x + y )( y + z )( x + z ) ≥1 Có lẽ tới bạn hiểu ta lại chuẩn hố Nhưng để tăng thêm niềm tin ta thử chuẩn hoá toán theo cách khác thử xem Chẳng hạn chuẩn hoá a + b + c = a b c Ta đặt a ' = ; b ' = ; c ' = ta cần chọn t cho a ' + b ' + c ' = , lúc ta tìmđược t = a + b + c t t t Bây ta xem chưa biết số t, ta tạo điều kiện a,b,c sau: (1) ab + bc + ca (a + b)(b + c)(c + a) ⇔ ≤3 3 (a + b + c) (a + b + c)  a  a + b   b + c  c + a  b b c c a   + + ≤      a +b +c a +b +c a +b +c a +b+c a +b+c a +b+c  a + b + c   a + b + c  a + b + c  a b c Đặt x = ;y= ;z = ta có điều kiện x + y + z = bất đẳng thức cần a +b+c a+b+c a+b+c xy + yz + xz ( x + y )( y + z )( x + z ) chứng minh trở thành: ≤ Để hiểu sau bạn đọc tự chuẩn hố tốn theo abc = (a + b)(b + c)(c + a) = Ghi chú: bất đẳng thức cịn có cách chứng minh hay sau: Sử dụng hai bất đẳng thức phụ sau: (a + b)(b + c )(c + a ) (a + b + c )(ab + bc + ca) ≥ (a + b + c) ≥ 3(ab + bc + ca ) Khi ta có: ⇔ (a + b)(b + c )(c + a ) (a + b + c )(ab + bc + ca) ≥ ≥ 3(ab + bc + ca )(ab + bc + ca ab + bc + ca = PDF created with pdfFactory Pro trial version www.pdffactory.com    ≥    Cách giải GV Hoàng Đức Nguyên-khối THT chuyên ĐHSP Hà Nội đưa chun mục “bạn đọc tìm tịi” báo Toán hoc tuổi trẻ với tên “Ứng dụng đẳng thức” Như ta thấy bất đẳng thức chuẩn hố nhiều cách khác Chuẩn hoá kĩ thuật kĩ thuật lại đòi hỏi kinh nghiệm độ tinh tế định Đây điều độc đáo khó khăn nhât kĩ thuật này, chuẩn hố cách hợp lí ta có lời giải toán đơn giản Bây xem thử kĩ thuật chuẩn hố có sức mạnh giới bất đẳng thức Bắt đầu từ trở toán ta khơng giải thích rõ ràng cách chuẩn hố mà điểm dành cho bạn đọc D Kĩ thuật chuẩn hoá ứng dụng: Trong phần tập cố gắng ghi rõ nguồn gốc xuất xứ toán từ đâu Tuy nhiên có số hạn chế nên có số tốn chúng tơi khơng ghi rõ nguồn gốc xuất xứ mong bạn đọc thông cảm Problem 1: ( England-1999) Cho x, y, z không âm Chứng minh rằng: ( x + y + z ) + xyz ≥ ( x + y + z )( xy + yz + xz ) Solution: Bất đẳng thức cho nên ta chuẩn hoá x + y + z = , bất đẳng thức cần chứng minh trở thành: + 9abc ≥ 7(ab + bc + ca) (1) Do tính đối xứng bất đẳng thức cần chứng minh nên ta hồn tồn giả sử x = max { x, y, z} 1  Ta xét f ( x) = (7 y + z − yz ) x + yz − với x ∈  ,1 3  Sử dụng Look at the end point (định lí 1) ta có: 2 2    7( y + z ) + 12 yz − 4(9 yz − 1)  y+z  3 f  = = ≤ yz ≤  = (2)  = 3  3       f (1) = 7( y + z ) − yz − = −2 < x = ⇒ y = z = (3) Từ (2)&(3) suy f ( x ) ≤ ⇒ bất đẳng thức cho Đẳng thức xảy ⇔ x = y = z Ta xét tiếp toán tương tự sau: Problem 2: ( Sưu tầm) Cho x, y, z không âm Chứng minh rằng: xyz + ( x + y + z )3 ≥ 4( xy + yz + xz )( x + y + z ) Solution: Bất đẳng thức cho nên ta chuẩn hoá x + y + z = Khi bất đẳng thức cần chứng minh trở thành: + xyz ≥ 4( xy + yz + xz ) Do bất đẳng thức có tính đối xứng nên ta giả sử x = max { x, y, z} PDF created with pdfFactory Pro trial version www.pdffactory.com 1  Xét f ( x ) = (4 y + z − yz ) x + yz − với x ∈  ,1 3  Sử dụng Look at the end point (định lí1) ta có: 2 2   y+z  3   4( y + z ) + yz − f  = = yz − ≤ yz ≤  = = (2)   3       f (1) = 4( y + z ) − yz − = −1 < x = ⇒ y = z = (3) Từ (2)&(3) suy ra: f ( x ) ≤ ⇒ bất đẳng thức cho Đẳng thức xảy ⇔ x = y = z Comment 1: Qua hai toán hẳn bạn đọc thấy hữu ích việc chuẩn hố Việc chuẩn hố khơng làm cho tốn nhìn đon giản mà cịn định hướng lời giải cho cách rõ ràng Quả thật toán từ trở sau viết ta khơng làm cơng việc chuẩn hố khó lời giải hay, đẹp toán Chú ý: Các toán điều chứng minh cách trực tiếp, cách khai triển hai vế rút gọn sau sử dụng thêm BĐT Schur Bằng phương pháp tương tự bạn đọc tự giải hai toán sau: Problem 3: (IMO-1984) Cho x, y, z không âm Chứng minh rằng: 27( x + y + z )( xy + yz + xz ) ≤ 7( x + y + z )3 + 54 xyz Problem 4: ( Sưu tầm) Cho x, y, z không âm Chứng minh rằng: 2( x + y + z )3 ≤ 9( x + y + z ) + 27 xyz ≤ 9( x + y + z )3 Gợi ý: Chuẩn hoá x + y + z = BĐT VT xảy biên BĐT VP xảy tâm Sau ta xét tiếp lớp tốn có mức độ khó khăn Problem 5: (Macedonia 1999) Cho a, b, c không âm Chứng minh rằng: abc(a + b + c) + (a + b + c ) ≥ 4abc 3(a + b + c ) Solution: Bất đẳng thức cho đồng thời với mong muốn biểu thức ta chuẩn hoá a + b + c = Khi bất đẳng thức cần chứng minh trở thành: abc(a + b + c ) + ≥ 3.abc ⇔ a + b + c + ≥ (1) abc Tới giải theo cách thông thường ta thu hai bất đẳng thức trái dấu là: PDF created with pdfFactory Pro trial version www.pdffactory.com a+b+c ≤ ≥3 abc Vì ta phải tìm cách khác để giải (1) Phương pháp tối ưu lúc sử dụng kĩ thuật chọn điểm rơi với bất đẳng thức AM-GM Ta tìm cách tách a + b + c để sử dụng bất đẳng thức AM-GM dấu xảy ra, đưa tham số α vào ta có: αa + αb + αc + + (1 − α )(a + b + c) abc tính đối xứng toán nên ta dự Sử dụng BĐT AM-GM dấu xảy α a = abc đoán dấu xảy biến nhau, nghĩa lúc ta có: α = = Như ta giải a (1) sau: 1 (1) ⇔ 9a + 9b + 9c + − 8(a + b + c) ≥ 4 93.abc − 3(a + b + c ) = 12 − = abc abc Vậy bất đẳng thức cho Đẳng thức xảy ⇔ a = b = c Comment 2: Bất đẳng thức AM-GM bất đẳng thức nên hữu hiệu việc chứng minh bất đẳng thức khác Tuy nhiên điều khó khăn điều kiện xảy dấu nghiêm ngặt, việc áp dụng trực tiếp cách máy móc dễ dẫn đến sai lầm Bất đẳng thức AM-GM có nhiều kĩ thuật sử dụng bạn đọc nên biết kĩ thuật chính: + Kĩ thuật cân hệ số: sử dụng để giải bất đẳng thức không đối xứng.(sẽ giới thiệu phần sau) + Kĩ thuật chọn điểm rơi-trọng số: sử dụng để giải bất đẳng thức đối xứng ta nhận thấy dấu xảy toán + Kĩ thuật AM-GM ngược dấu: sử dụng để giải bất đẳng thức hoán vị Sử dụng kĩ thuật ta giải toán sau: Problem 6: (Crux 2946) Cho a, b, c dương Chứng minh rằng: (ab + bc + ca)(a + b + c ) + abc (a + b + c) ≥ 4abc 3(a + b + c ) Solution: Bất đẳng thức cho với mong muốn làm biểu thức nên ta chuẩn hoá a + b + c = Khi bất đẳng thức cần chứng minh trở thành: ab + bc + ca + abc (a + b + c) ≥ 3.abc 1 ⇔ a + b + c + + + ≥ 3(1) a b c Sử dụng kĩ thuật chọn điểm rơi ta có: 1 1 (1) ⇔ 3a + 3b + 3c + + + − 2(a + b + c) ≥ 6 33.abc − 3(a + b + c ) = − = a b c abc Vậy bất đẳng thức chứng minh Đẳng thức xảy ⇔ a = b = c PDF created with pdfFactory Pro trial version www.pdffactory.com Problem 7: (Sưu tầm) Cho a, b, c dương Chứng minh rằng: (ab + bc + ca)(a + b + c ) ≥ 2abc 3(a + b + c ) + abc (a + b + c) Solution: Bất đẳng thức cho ta chuẩn hoá a + b + c = Khi bất đẳng thức cần chứng minh trở thành: ab + bc + ca ≥ 3.abc + abc(a + b + c ) 1 ⇔ + + − (a + b + c ) ≥ 3(1) a b c Sử dụng kĩ thuật chọn điểm rơi ta có: 1 1 (1) ⇔ + + + 3a + 3b + 3c − 4(a + b + c) ≥ 33.abc − 3(a + b + c ) = − = a b c abc Vậy bất đẳng thức cho Đẳng thức xảy ⇔ a = b = c Bạn đọc tự luyện sau Problem 8: (Sưu tầm) Cho a, b, c dương Chứng minh rằng: 2(ab + bc + ca )(a + b + c ) + 3abc(a + b + c ) ≥ 15abc ( a + b + c ) Problem 9: (Sưu tầm) Cho a, b, c dương Chứng minh rằng: 3(ab + bc + ca )(a + b + c ) + 4abc(a + b + c ) ≥ 21abc ( a + b + c ) Problem 10: (Sưu tầm) Cho a, b, c dương; m,n dương 2n ≥ m Chứng minh rằng: mabc(a + b + c) + n(ab + bc + ca)(a + b + c ) ≥ 3(m + n)abc ( a + b + c ) Hướng dẫn: NX: Đây toán tổng quát toán Sau chuẩn hoá a + b + c = ta có bất đẳng thức sau: 1 1 m(a + b + c ) + n  + +  ≥ 3(m + n) (*) a b c Ngoài cách sử dụng kĩ thuật chọn điểm rơi, ta sử dụng cách giải sau: 1 Đặt X = + + ; Y = a + b + c a b c PDF created with pdfFactory Pro trial version www.pdffactory.com M (a, b, c ) M (b, c, a) M (c, a, b) + + − 3t = G (a, b, c)(b − c) + G (b, c, a)(c − a )2 + G (c, a, b)(a − b) N (a, b, c) N (b, c, a ) N (c, a, b) Bổ đề 4: (Biểu diễn tắc) Giả sử α , β , γ hửu tỉ có tổng 3k , tồn biểu diễn sở cho biểu thức f k (a, b, c) = ∑ aα ′b β ′cγ ′ − 6a k b k c k F (a, b, c) = sym Trong tổng lấy tất hoán vị (α ′, β ′, γ ′ ) (α , β , γ ) Định lí S.O.S: Xét biểu thức S = f (a, b, c ) = Sa (b − c )2 + Sb (a − c )2 + Sc (a − b)2 Trong S a , Sb , Sc hàm số a, b, c Nếu S a , Sb , Sc ≥ S ≥ Nếu a ≥ b ≥ c Sb , Sb + Sc , Sb + S a ≥ S ≥ Nếu a ≥ b ≥ c S a , Sc , Sa + 2Sb , S c + Sb ≥ S ≥ Nếu a ≥ b ≥ c Sb , S c ≥ 0, a Sb + b Sa ≥ S ≥ Nếu S a + Sb + Sc ≥ S a Sb + Sb Sc + Sc Sa ≥ S ≥ Ngồi để S ≥ với a, b, c ≥ ta phải có: S a + Sb |a =b ≥ 0, Sb + Sc |b =c ≥ 0, Sc + Sa |c= a ≥ S a + Sb |a =b ≥ có nghĩa ta xét biểu thức S a + Sb a = b Với toán đối xứng ta có S a = Sb a = b Nhận xét quan trọng tốn tìm số tốt Sau số đẳng thức thường dùng phân tích: 1.a + b − 2ab = (a − b)2 ( a − b) a b + − = b a ab 3 3.a + b − ab(a + b) = (a + b)(a − b) 4.a + b + c − ab − bc − ca =  (a − b)2 + (b − c )2 + (c − a )2  2 5.a + b + c − (a + b + c) = (a − b)2 + (b − c) + (c − a )2  3 6.(a + b)(b + c)(c + a) − 8abc = a (b − c )2 + b(c − a )2 + c(a − b)2 7.a + b3 + c3 − 3abc = (a + b + c )  (a − b) + (b − c )2 + (c − a )2  ( a − b) 2(a + b ) − (a + b) = a + b + 2(a + b ) 1 (a − b)2 ( (a + b)2 + c ) + (b − c)2 ( (b + c) + a ) + (c − a )2 ( (c + a) + b )  a b c ( a − b) (b − c) ( c − a) + + − = + + 10 b + c c + a a + b 2(a + c )(b + c) 2(a + b)(a + c) 2(b + c)(b + a ) * Chú ý: phương pháp phân tích khơng Ta hiểu "nơm na" phương pháp phân tích bình phương S.O.S sau: 9.a + b + c − abc(a + b + c ) = PDF created with pdfFactory Pro trial version www.pdffactory.com Giả sử ta cần chứng minh bất đẳng thức A ≥ B ta biết chứng minh được: A ≥ C A ≤ C ∨  B ≥ C  B ≤ C Khi thay ta chứng minh bất đẳng thức cho ta qui vế chứng minh bất đẳng thức: A − C ≥ B − C (1) C − A ≤ C − B (2) Lợi ích phương pháp là: Do vế bất đẳng thức (1) , (2) không âm (thông thường đại lượng bình phương đủ) Khi việc ta cần làm so sánh hệ số đại lượng khơng âm với D Một số tốn khơng cần sử dụng tiêu chuẩn: Problem 1: (Trần Nam Dũng) Cho a, b, c dương Chứng minh rằng: 1+ a + b2 + c a b c ≥ + + ab + bc + ca b + c c + a a + b Lời giải: * Ta nhận thấy hai vế lớn hoăc 3/2 Bất đẳng thức cần chứng minh tương đương với bất đẳng thức sau: a2 + b2 + c a −1 ≥ ∑ − ab + bc + ca cyc b + c ⇔ ∑ ( a − b) cyc ab + bc + ca ≥∑ cyc ( a − b) 2(a + c)(b + c) Đưa bất đẳng thức dạng tắc: S a (b − c )2 + Sb (c − a )2 + S c (a − b) ≥ Sa = a2 >0 (b + a)(c + a) Trong đó: Sb = b2 >0 (b + a )(b + c ) c2 >0 (c + a)(c + b) Vậy toán chứng minh Đẳng thức xảy ⇔ a = b = c Sc = Problem 2: (Sưu tầm) Cho a, b, c dương Chứng minh rằng: a2 + ∑ a ≥ 3.∑ a ∑ cyc b cyc cyc Lời giải: Ta nhận thấy hai vế bất đẳng thức lớn bằng: 2(a + b + c) Bất đẳng thức cần chứng minh tương đương với bất đẳng thức sau: PDF created with pdfFactory Pro trial version www.pdffactory.com a2 − ∑ a ≥ 3.∑ a − 2∑ a ∑ b cyc cyc cyc cyc ⇔∑ cyc ∑ ( a − b) ( a − b) ≥ cyc b 3.∑ a + ∑ a cyc cyc Đưa bất đẳng thức cần chứng minh dạng tắc: S a (b − c )2 + Sb (c − a )2 + S c (a − b) ≥ 1 a +b Sa = − ≥ − = >0 c 3.∑ a + ∑ a c a + b + c c(a + b + c) cyc Trong đó: Sb = cyc 1 b+c − ≥ − = >0 a 3.∑ a + ∑ a a a + b + c a (a + b + c) cyc Sc = cyc 1 a+c − ≥ − = >0 b 3.∑ a + ∑ a b a + b + c b(a + b + c) cyc Vậy bất đẳng thức chứng minh Đẳng thức xảy ⇔ a = b = c Sau số tự luyện dành cho bạn đọc bước đầu làm quen với phương pháp Problem 3: (Crux 2290)   Cho a, b, c dương Chứng minh rằng:  ∏ (a + b)  ≥ abc.∏ (2a + b + c ) cyc  cyc  Problem 4: (Sưu tầm)   Cho a, b, c dương Chứng minh rằng:  ∑ a  + ∑ a 2b ≥ 2∑ a 3b sym  cyc  cyc Problem 5:(Sưu tầm) a Cho a, b, c dương Chứng minh rằng: ∑ a ∑ + 15 ≥ 4∑ cyc cyc a sym b Problem 6:(Sưu tầm) 1 Cho a, b, c dương Chứng minh rằng: ∑ a ∑ + 27 ≥ 4∑ a.∑ cyc cyc a cyc cyc a Problem 7:(Sưu tầm) Cho a, b, c dương Chứng minh rằng: a2 1 ∑ + ∑ a ≥ 3.∑ a 2 cyc cyc b + c cyc E Một số toán sử dụng tiêu chuẩn: a2 ∑ + ∑a ≥ 3.∑ a cyc b + c cyc cyc Problem 3: (Bất đẳng thức Schur) Cho a, b, c dương Chứng minh rằng: ∑a cyc + 3abc ≥ ∑ ab 2(a + b ) cyc PDF created with pdfFactory Pro trial version www.pdffactory.com Lời giải: Đây toán mà phần trước mà ta đề cập tới Bây ta giải toán phương pháp S.O.S Ta nhận thấy hai vế bất đẳng thức lớn ∑ ab(a + b) Bất đẳng thức cần chứng minh cyc tương đương với : ∑a cyc + 3abc − ∑ ab(a + b) ≥ ∑ ab 2(a + b ) − ∑ ab(a + b) cyc ⇔∑ cyc cyc cyc (a + b − c)(a − b) ab(a − b) ≥∑ cyc 2(a + b ) + a + b 2 Đưa bất đẳng thức dạng tắc : S a (b − c )2 + Sb (c − a )2 + S c (a − b) ≥ bc (b + c − a ) Sa = − 2(b + c ) + b + c Trong đó: Sb = c+a−b ca − 2 2(c + a ) + a + c a+b−c ab − 2(a + b ) + a + b Không tính tổng quát giả sử a ≥ b ≥ c Khi đó: Sc = Sb = Sc = (c + a − b) ( ( ( a + b − c) ( 2(a + c ) + a + c ( ) 2(a + c ) + a + c − 2ac ) ) 2(c + a − b)(a + c) − 2ac ) 2(a + b2 ) + a + b − 2ab 2(a + b ) + a + b ≥ ≥ ( ) 2(a + c ) + a + c 2(a + b − c )(a + b) − 2ab ( 2(a + b ) + a + b ) ≥ ( c + a − b ≥ c; a + c ≥ a ) ≥ ( a + b − c ≥ a; a + b ≥ b ) ⇒ Sb + Sc ≥ Ta chứng minh :  a c  b   b ≥ 2b 1 − − = b +  ≥  a+b b+c 2(a + b ) + a + b 2(b + c ) + b + c  2(a + b) 2(b + c)  Vậy bất đẳng thức chứng minh Đẳng thức xảy ⇔ a = b = c ∨ a = b, c = hoán vị S a + Sc = 2b − 2ab − 2bc Problem 4: (Sưu tầm) Cho a, b, c dương Chứng minh rằng: a2 + b2 + c 8abc + ≥2 ab + bc + ca (a + b)(b + c )(c + a ) Lời giải: Như nói phần trước bất đẳng thức trênđược tạo nhờ phép cộng ngược chiều Ta chứng minh phương pháp S.O.S: Bất đẳng thức cần chứng minh tương đương với bất đẳng thức sau: PDF created with pdfFactory Pro trial version www.pdffactory.com ∑a ∑ ( a − b) 8abc −1 ≥ − ⇔ ∑ ab ∑ ab ∏ ( a + b) cyc cyc cyc cyc cyc ∑ 2c(a − b) ≥ ∏ (a + b ) cyc cyc Đưa bất đẳng thức cần chứng minh dạng tắc: S a (b − c )2 + Sb (c − a )2 + S c (a − b) ≥ (a + b)(b + c)(c + a) abc − 2b = b + c − a − ab + bc + ca ab + bc + ca (a + b)(b + c )(c + a ) abc − 2b = a + c − b − Trong đó: Sb = ab + bc + ca ab + bc + ca (a + b)(b + c )(c + a ) abc Sc = − 2c = a + b − c − ab + bc + ca ab + bc + ca Khơng tính tổng qt giả sử a ≥ b ≥ c Khi dễ thấy Sb , Sc ≥ ⇒ Sb + S c ≥ Sa = 2abc 2c (a + b) = ≥ Ta chứng minh: S a + Sc = 2c − ab + bc + ca ab + bc + ca Vậy bất đẳng thức chứng minh Đẳng thức xảy ⇔ a = b = c ∨ a = b, c = hoán vị Comment 1: Bài tốn đặt tìm số k tốt cho bất đẳng thức sau với a, b, c không âm a + b2 + c2 8abc k + ≥ + k (Sẽ xét phần sau) ab + bc + ca (a + b)(b + c)(c + a) Problem 5: (Sưu tầm) Cho a, b, c dương Chứng minh rằng: 3 a2 3(a + b + c ) ≥ ∑ a + b2 + c cyc b Lời giải: Ta thấy hai vế bất đẳng thức lớn bằng: ∑ a Bất đẳng thức cần chứng minh tương đương cyc a2 −∑a ≥ ∑ cyc b cyc 3.∑ a cyc ∑a −∑a ⇔ ∑ cyc cyc ( a − b) (a + b)(a − b)2 ≥∑ b a + b2 + c2 cyc cyc Đưa bất đẳng thức chứng minh vế dạng tắc: S a (b − c )2 + Sb (c − a )2 + S c (a − b) ≥ b2 + a2 −b c c2 + b2 Trong đó: Sb = −c a a2 + c Sc = −a b Do bất đẳng thức có tính hốn vị ta xét hai trường hợp: Sa = PDF created with pdfFactory Pro trial version www.pdffactory.com * a ≥ b ≥ c Khi ta có: S a , Sc ≥ Ta chứng minh: S a + Sb ≥ 0; Sc + Sb ≥ hai bất đẳng thức chứng minh khơng khó, dành cho bạn đọc * a ≤ b ≤ c Khi ta có: Sb , S c ≥ Dễ dàng chứng minh: S a + Sb ≥ Vậy bất đẳng thức chứng minh Đẳng thức xảy ⇔ a = b = c Bây ta xét số toán với yêu cầu tìm số k để bất đẳng thức thoả mãn đềi kiện Ta bắt đầu với toán sau: Problem 6: (Nguyễn Anh Khoa) Tìm số dương k tốt để bất đẳng thức sau đúng: a + b2 + c2 8abc k + ≥ 1+ k ab + bc + ca (a + b)(b + c)(c + a) a, b, c số thực không âm tuỳ ý Lời giải: Bất đẳng thức cần chứng minh tương đương với bất đẳng thức sau:  a2 + b2 + c  8abc k  − 1 ≥ − (a + b)(b + c)(c + a)  ab + bc + ca  ⇔ k ∑ ( a − b) cyc ab + bc + ca ≥ ∑ c ( a − b) cyc (a + b)(b + c)(c + a) Đưa bất đẳng thức dạng tắc: S a (b − c )2 + Sb (c − a )2 + S c (a − b) ≥ Trong hệ số xác định bởi: S a = k (a + b)(b + c )(c + a ) − a(ab + bc + ca ) Sb = k (a + b)(b + c)(c + a ) − b(ab + bc + ca) S c = k (a + b)(b + c)(c + a ) − c(ab + bc + ca ) ĐK cần: Lấy b = c , ta có Sb = Sc Đế bất đẳng thức phải có: 2ab + b Sb + Sc |b =c = Sb ≥ ⇔ k ≥ 2(a + b) 1 Mặt khác max f (a, b) = Từ ta suy giá trị tốt k 2 ĐK đủ: Với k = Khơng tính tổng quát giả sử a ≥ b ≥ c ⇒ Sa ≤ Sb ≤ Sc (a + c − b) abc Sb = (ab + bc + ca) − ≥ (a + c − b ≥ c) 2 (a + b − c ) abc S c = (ab + bc + ca) − ≥ 0(a + b − c ≥ a) 2 ⇒ Sb + Sc ≥ S a + Sb = (a + b)(b + c )(c + a ) − (a + b)(ab + bc + ca ) = (a + b)c ≥ Vậy bất đẳng thức với a, b, c ≥ ⇔ k ≥ Đẳng thức xảy ⇔ a = b = c ∨ a = b, c = hoán vị PDF created with pdfFactory Pro trial version www.pdffactory.com Problem 7: (Bùi Việt Anh) Tìm số dương k tốt để bất đẳng thức sau đúng: a + b3 + c abc k + k ≥ 3+ abc 27 (a + b + c) a, b, c số thực không âm tuỳ ý Lời giải: Bất đẳng thức cần chứng minh tương đương với bất đẳng thức sau:   a + b3 + c abc −3 ≥ k  −  abc  27 (a + b + c)  ⇔ (a + b + c)∑ (a − b) (a + b + 7c)∑ (a − b) ≥ k 2abc 54(a + b + c)3 Đưa bất đẳng thức dạng tắc: S a (b − c )2 + Sb (c − a )2 + S c (a − b) ≥ Trong hệ số xác định bởi: S a = 27(a + b + c) − kabc (7a + b + c) cyc cyc Sb = 27(a + b + c )4 − kabc(a + 7b + c ) S c = 27(a + b + c)4 − kabc (a + b + 7c ) ĐK cần: Lấy b = c , ta có: Sb = Sc Để bất đẳng thức ta phải có: Sb + Sc |b =c = Sb ≥ ⇔ k ≤ Mặt khác f (a, b) = 27(a + 2b)4 ab (a + 8b) 729 729 Từ suy giá trị tốt k 4 729 Khơng tính tổng qt giả sử a ≥ b ≥ c ⇒ Sa ≤ Sb ≤ Sc Sb = 4(a + b + c)4 − 27 abc(a + 7b + c ) ≥ 27 abc(4a + 4b + 4c − a − 7b − c) = 27abc (3a − 3b + 3c) ≥ ĐK đủ: Với k = S c = 4(a + b + c) − 27 abc(a + b + 7c ) ≥ 27 abc(4a + 4b + 4c − a − b − 7c ) = 27abc (3a + 3b − 3c) ≥ ⇒ Sb + Sc ≥ S a + Sb = 4(a + b + c )4 − 27 abc(4a + 4b + c) ≥ 27abc (4a + 4b + 4c − 4a − 4b − c ) ≥ 729 Vậy bất đẳng thức với a, b, c ≥ ⇔ k ≤ Đẳng thức xảy ⇔ a = b = c Sau số tập tự luyện: Problem 1:(Sưu tầm) Cho a, b, c dương Chứng minh rằng: a b c abc + + + ≥ 3 b + c c + a a + b 2(a + b + c ) Problem 2:(Sưu tầm) Cho a, b, c dương Chứng minh rằng: 1 21 + + ≥ 2 2 2 a + ab + b b + bc + c c + ca + a 2(a + b + c ) + 5(ab + bc + ca ) PDF created with pdfFactory Pro trial version www.pdffactory.com 1 27 + + ≥ 2 8a + 7ab + 8b 8b + 7bc + 8c 8c + 7ca + 8a 23(a + b + c)2 Problem 2:(Phạm Kim Hùng ) Tìm số dương k để bất đẳng thức sau với a, b, c ≥ : (a + b)(b + c )(c + a ) ab + bc + ca + k ≥ 8+ k abc a + b2 + c2 ĐS: k ≤ Problem 4:(Sưu tầm) Tìm số dương k để bất đẳng thức sau với a, b, c ≥ : a b c ab + bc + ca + + + k ≥k+ 2 b+c c+a a +b a +b +c −1 ĐS: k ≤ Lưu ý: Các toán đơn giản giúp bạn lần đầu tiếp cận phương pháp S.O.S Phương pháp mạnh toán bất đẳng thức đối xứng biến dạng đa thức, phân thức, khơng có thức (vẫn có trường hợp ngoại lệ cụ thể Problem 2), hầu hết bất đẳng thức loại phải "chịu thua" trước sức mạnh Để nắm rõ điêu luyện phương pháp bạn đọc nên tham khảo thêm sách "Sáng tạo bất đẳng thức" tác giả Phạm Kim Hùng truy cập vào trang web giới thiệu phần mục lục 2 Lời kết: Bất đẳng thức giới đẹp đẽ biết bao, sâu vào ta cảm nhận hết vẻ đẹp mà mang lại Như bạn biết bất đẳng thức phần rộng có nhiều đề tài hấp dẫn chưa nghiên cứu Cũng lí đồng thời tơi học lớp 10 nên kiến thức có hạn khó tránh khỏi sai sót, cố gắng hoán thành viết thời gian ngắn Tác giả xin chân thành cảm ơn đóng góp ý kiến chân thành bạn đọc viết PDF created with pdfFactory Pro trial version www.pdffactory.com PDF created with pdfFactory Pro trial version www.pdffactory.com PDF created with pdfFactory Pro trial version www.pdffactory.com PDF created with pdfFactory Pro trial version www.pdffactory.com PDF created with pdfFactory Pro trial version www.pdffactory.com PDF created with pdfFactory Pro trial version www.pdffactory.com PDF created with pdfFactory Pro trial version www.pdffactory.com PDF created with pdfFactory Pro trial version www.pdffactory.com

Ngày đăng: 23/05/2021, 02:31

Xem thêm:

TỪ KHÓA LIÊN QUAN

w